You are on page 1of 15

Actual CAT Problems 1998-2006

Number System
CAT 1998
1. n3 is odd. Which of the following statement(s) is(are) true?
I. n is odd. II. n2 is odd. III. n2 is even.
a. I only b. II only c. I and II d. I and III

2. (BE)2 = MPB, where B, E, M and P are distinct integers. Then M =


a. 2 b. 3 c. 9 d. None of these

3. Five-digit numbers are formed using only 0, 1, 2, 3, 4 exactly once. What is the difference between
the maximum and minimum number that can be formed?
a. 19800 b. 41976 c. 32976 d. None of these

4. A certain number, when divided by 899, leaves a remainder 63. Find the remainder when the same
number is divided by 29.
a. 5 b. 4 c. 1 d. Cannot be determined

5. A is the set of positive integers such that when divided by 2, 3, 4, 5, 6 leaves the remainders 1, 2, 3,
4, 5 respectively. How many integers between 0 and 100 belong to set A?
a. 0 b. 1 c. 2 d. None of these

6. How many five-digit numbers can be formed using the digits 2, 3, 8, 7, 5 exactly once such that the
number is divisible by 125?
a. 0 b. 1 c. 4 d. 3

7. What is the digit in the unit’s place of 251?


a. 2 b. 8 c. 1 d. 4

8. A number is formed by writing first 54 natural numbers in front of each other as 12345678910111213
... Find the remainder when this number is divided by 8.
a. 1 b. 7 c. 2 d. 0

CAT 1999

9. Let a, b, c be distinct digits. Consider a two-digit number ‘ab’ and a three-digit number ‘ccb’, both
defined under the usual decimal number system, if (ab)2 = ccb > 300, then the value of b is
a. 1 b. 0 c. 5 d. 6

10. The remainder when 784 is divided by 342 is


a. 0 b. 1 c. 49 d. 341

Number System Page 1


11. If n = 1 + x where x is the product of four consecutive positive integers, then which of the following
is/are true?
A. n is odd
B. n is prime
C. n is a perfect square
a. A and C only b. A and B only c. A only d. None of these

12. For two positive integers a and b define the function h(a,b) as the greatest common factor (G.C.F) of
a, b. Let A be a set of n positive integers. G(A), the G.C.F of the elements of set A is computed by
repeatedly using the function h. The minimum number of times h is required to be used to compute
G is
1
a. n b. (n – 1) c. n d. None of these
2

13. If n2 = 12345678987654321, what is n?


a. 12344321 b. 1235789 c. 111111111 d. 11111111
Directions for questions 14 to 16: Answer the questions based on the following information.
There are 50 integers a1, a2 … a50, not all of them necessarily different. Let the greatest integer of these 50
integers be referred to as G, and the smallest integer be referred to as L. The integers a1 through a24 form
sequence S1, and the rest form sequence S2. Each member of S1 is less than or equal to each member
of S2.
14. All values in S1 are changed in sign, while those in S2 remain unchanged. Which of the following
statements is true?
a. Every member of S1 is greater than or equal to every member of S2.
b. G is in S1.
c. If all numbers originally in S1 and S2 had the same sign, then after the change of sign, the
largest number of S1 and S2 is in S1.
d. None of the above

15. Elements of S1 are in ascending order, and those of S2 are in descending order. a24 and a25 are
interchanged. Then which of the following statements is true?
a. S1 continues to be in ascending order.
b. S2 continues to be in descending order.
c. S1 continues to be in ascending order and S2 in descending order.
d. None of the above

16. Every element of S1 is made greater than or equal to every element of S2 by adding to each element
of S1 an integer x. Then x cannot be less than
a. 210
b. the smallest value of S2
c. the largest value of S2
d. (G – L)

Page 2 Number System


CAT 2000

17. Let D be a recurring decimal of the form D = 0. a1 a2 a1 a2 a1 a2 ..., where digits a1 and a2 lie between
0 and 9. Further, at most one of them is zero. Which of the following numbers necessarily produces
an integer, when multiplied by D?
a. 18 b. 108 c. 198 d. 288

18. Consider a sequence of seven consecutive integers. The average of the first five integers is n. The
average of all the seven integers is
a. n b. n + 1
2
c. k × n, where k is a function of n d. n +  
7

19. Let S be the set of integers x such that


I. 100 ≤ x ≤ 200 ,
II. x is odd and
III. x is divisible by 3 but not by 7.
How many elements does S contain?
a. 16 b. 12 c. 11 d. 13

20. Let x, y and z be distinct integers, that are odd and positive. Which one of the following statements
cannot be true?
a. xyz2 is odd b. (x – y)2 z is even
2
c. (x + y – z) (x + y) is even d. (x – y)(y + z)(x + y – z) is odd

21. Let S be the set of prime numbers greater than or equal to 2 and less than 100. Multiply all elements
of S. With how many consecutive zeros will the product end?
a. 1 b. 4 c. 5 d. 10

22. What is the number of distinct triangles with integral valued sides and perimeter 14?
a. 6 b. 5 c. 4 d. 3

23. Let N = 1421 × 1423 × 1425. What is the remainder when N is divided by 12?
a. 0 b. 9 c. 3 d. 6

24. The integers 34041 and 32506, when divided by a three-digit integer n, leave the same remainder.
What is the value of n?
a. 289 b. 367 c. 453 d. 307

25. Each of the numbers x1, x 2 , L, x n , n ≥ 4, is equal to 1 or –1. Suppose


x1x 2 x 3 x 4 + x 2 x 3 x 4 x 5 + x 3 x 4 x 5 x 6 + L + x n −3 x n − 2 x n −1x n + x n −2 x n −1x n x1 + x n −1x n x1x 2 + x n x1x 2 x 3 = 0,
then
a. n is even b. n is odd
c. n is an odd multiple of 3 d. n is prime

Number System Page 3


26. Sam has forgotten his friend’s seven-digit telephone number. He remembers the following: the first
three digits are either 635 or 674, the number is odd, and the number 9 appears once. If Sam were
to use a trial and error process to reach his friend, what is the minimum number of trials he has to
make before he can be certain to succeed?
a. 10,000 b. 2,430
c. 3,402 d. 3,006

27. Let N = 553 + 173 – 723. N is divisible by


a. both 7 and 13 b. both 3 and 13 c. both 17 and 7 d. both 3 and 17

28. Convert the number 1982 from base 10 to base 12. The result is
a. 1182 b. 1912 c. 1192 d. 1292

CAT 2001

29. Let x, y and z be distinct integers. x and y are odd and positive, and z is even and positive. Which
one of the following statements cannot be true?
a. y(x – z)2 is even b. y2(x – z) is odd c. y(x – z) is odd d. z(x – y)2 is even

30. In a four-digit number, the sum of the first 2 digits is equal to that of the last 2 digits. The sum of the
first and last digits is equal to the third digit. Finally, the sum of the second and fourth digits is twice
the sum of the other 2 digits. What is the third digit of the number?
a. 5 b. 8 c. 1 d. 4

31. Anita had to do a multiplication. In stead of taking 35 as one of the multipliers, she took 53. As a
result, the product went up by 540. What is the new product?
a. 1050 b. 540 c. 1440 d. 1590

32. x and y are real numbers satisfying the conditions 2 < x < 3 and – 8 < y < –7. Which of the following
expressions will have the least value?
a. x2y b. xy2 c. 5xy d. None of these

33. In a number system the product of 44 and 11 is 3414. The number 3111 of this system, when
converted to the decimal number system, becomes
a. 406 b. 1086 c. 213 d. 691

34. All the page numbers from a book are added, beginning at page 1. However, one page number was
added twice by mistake. The sum obtained was 1000. Which page number was added twice?
a. 44 b. 45 c. 10 d. 12

35. If a, b, c and d are four positive real numbers such that abcd = 1, what is the minimum value of
(1 + a)(1 + b)(1 + c)(1 + d)?
a. 4 b. 1 c. 16 d. 18

36. A set of consecutive positive integers beginning with 1 is written on the blackboard. A student came
7
along and erased one number. The average of the remaining numbers is 35 . What was the
17
number erased?
a. 7 b. 8 c. 9 d. None of these

Page 4 Number System


37 Let b be a positive integer and a = b2 – b. If b ≥ 4 , then a2 – 2a is divisible by
a. 15 b. 20 c. 24 d. All of these

38. In some code, letters a, b, c, d and e represent numbers 2, 4, 5, 6 and 10. We just do not know
which letter represents which number. Consider the following relationships:
I. a + c = e, II. b – d = d and III. e + a = b
Which of the following statements is true?
a. b = 4, d = 2 b. a = 4, e = 6 c. b = 6, e = 2 d. a = 4, c = 6

39. Let n be the number of different five-digit numbers, divisible by 4 with the digits 1, 2, 3, 4, 5 and 6, no
digit being repeated in the numbers. What is the value of n?
a. 144 b. 168 c. 192 d. None of these

CAT 2002

40. If there are 10 positive real numbers n1 < n2 < n3 ... < n10 , how many triplets of these numbers

(n1, n2 ,n3 ), (n2 , n3 ,n4 ), ... can be generated such that in each triplet the first number is always
less than the second number, and the second number is always less than the third number?
a. 45 b. 90 c. 120 d. 180

41. Number S is obtained by squaring the sum of digits of a two-digit number D. If difference between
S and D is 27, then the two-digit number D is
a. 24 b. 54 c. 34 d. 45

42. A rich merchant had collected many gold coins. He did not want anybody to know about him. One
day, his wife asked, " How many gold coins do we have?" After a brief pause, he replied, "Well! if I
divide the coins into two unequal numbers, then 48 times the difference between the two numbers
equals the difference between the squares of the two numbers." The wife looked puzzled. Can you
help the merchant's wife by finding out how many gold coins the merchant has?
a. 96 b. 53 c. 43 d. None of these

43. A child was asked to add first few natural numbers (i.e. 1 + 2 + 3 + …) so long his patience
permitted. As he stopped, he gave the sum as 575. When the teacher declared the result wrong, the
child discovered he had missed one number in the sequence during addition. The number he missed
was
a. less than 10 b. 10 c. 15 d. more than 15

44. When 2256 is divided by 17, the remainder would be


a. 1 b. 16 c. 14 d. None of these

45. At a bookstore, ‘MODERN BOOK STORE’ is flashed using neon lights. The words are individually
1 1 1
flashed at the intervals of 2 s, 4 s and 5 s respectively, and each word is put off after a second.
2 4 8
The least time after which the full name of the bookstore can be read again is
a. 49.5 s b. 73.5 s c. 1744.5 s d. 855 s

Number System Page 5


1 3 1
46. Three pieces of cakes of weights 4 lb, 6 lb and 7 lb respectively are to be divided into parts of
2 4 5
equal weight. Further, each part must be as heavy as possible. If one such part is served to each
guest, then what is the maximum number of guests that could be entertained?
a. 54 b. 72 c. 20 d. None of these

47. After the division of a number successively by 3, 4 and 7, the remainders obtained are 2, 1 and 4
respectively. What will be the remainder if 84 divides the same number?
a. 80 b. 75 c. 41 d. 53

48. If u, v, w and m are natural numbers such that um + vm = w m , then which one of the following is
true?
a. m ≥ min(u, v, w) b. m ≥ max(u, v, w)
c. m < min(u, v, w) d. None of these

49. 76n – 66n , where n is an integer > 0, is divisible by


a. 13 b. 127 c. 559 d. All of these

50. How many numbers greater than 0 and less than a million can be formed with the digits 0, 7
and 8?
a. 486 b. 1,084 c. 728 d. None of these

CAT 2003 (Leaked Paper)

51. How many even integers n, where 100 ≤ n ≤ 200 , are divisible neither by seven nor by nine?
a. 40 b. 37 c. 39 d. 38

52. A positive whole number M less than 100 is represented in base 2 notation, base 3 notation, and
base 5 notation. It is found that in all three cases the last digit is 1, while in exactly two out of the
three cases the leading digit is 1. Then M equals
a. 31 b. 63 c. 75 d. 91

Directions for question 53: Each question is followed by two statements, A and B. Answer each
question using the following instructions.
Choose (a) if the question can be answered by one of the statements alone but not by the other.
Choose (b) if the question can be answered by using either statement alone.
Choose (c) if the question can be answered by using both the statements together, but cannot be answered
by using either statement alone.
Choose (d) if the question cannot be answered even by using both the statements together.

53. Is a44 < b11, given that a = 2 and b is an integer?


A. b is even
B. b is greater than 16

54. How many three digit positive integers, with digits x, y and z in the hundred's, ten's and unit's place
respectively, exist such that x < y, z < y and x ≠ 0 ?
a. 245 b. 285 c. 240 d. 320

Page 6 Number System


55. If the product of n positive real numbers is unity, then their sum is necessarily
1
a. a multiple of n b. equal to n + c. never less than n d. a positive integer
n

56. The number of positive integers n in the range 12 ≤ n ≤ 40 such that the product (n − 1)(n − 2)K 3.2.1
is not divisible by n is
a. 5 b. 7 c. 13 d. 14

CAT 2003 (Re test)

Directions for questions 57 to 59: Answer the questions on the basis of the information given below.
The seven basic symbols in a certain numeral system and their respective values are as follows:
I = 1, V = 5, X = 10, L = 50, C = 100, D = 500 and M = 1000
In general, the symbols in the numeral system are read from left to right, starting with the symbol
representing the largest value; the same symbol cannot occur continuously more than three times; the
value of the numeral is the sum of the values of the symbols. For example, XXVII = 10 + 10 + 5 + 1 + 1
= 27. An exception to the left-to-right reading occurs when a symbol is followed immediately by a
symbol of greater value; then the smaller value is subtracted from the larger.
For example, XLVI = (50 – 10) + 5 + 1 = 46.

57. The value of the numeral MDCCLXXXVII is


a. 1687 b. 1787 c. 1887 d. 1987

58. The value of the numeral MCMXCIX is


a. 1999 b. 1899 c. 1989 d. 1889

59. Which of the following represent the numeral for 1995?


I. MCMLXXV II. MCMXCV III. MVD IV. MVM

a. Only I and II b. Only III and IV c. Only II and IV d. Only IV

60. What is the sum of all two-digit numbers that give a remainder of 3 when they are divided by 7?
a. 666 b. 676 c. 683 d. 77

61. An intelligence agency forms a code of two distinct digits selected from 0, 1, 2, …, 9 such that the
first digit of the code is non-zero. The code, handwritten on a slip, can however potentially create
confusion when read upside down — for example, the code 91 may appear as 16. How many codes
are there for which no such confusion can arise?
a. 80 b. 78 c. 71 d. 69

62. What is the remainder when 496 is divided by 6?


a. 0 b. 2 c. 3 d. 4

63. Let n (>1) be a composite integer such that n is not an integer. Consider the following statements:

A: n has a perfect integer-valued divisor which is greater than 1 and less than n
B: n has a perfect integer-valued divisor which is greater than n but less than n
a. Both A and B are false b. A is true but B is false
c. A is false but B is true d. Both A and B are true

Number System Page 7


64. If a, a + 2 and a + 4 are prime numbers, then the number of possible solutions for a is
a. one b. two c. three d. more than three

65. Let a, b, c, d and e be integers such that a = 6b = 12c, and 2b = 9d = 12 e. Then which of the
following pairs contains a number that is not an integer?
 a b  a c  a bd  a c
a.  ,  b.  ,  c.  ,  d.  , 
 27 e   36 e   12 18   6 d

CAT 2004

66. On January 1, 2004 two new societies s1 and s2 are formed, each n numbers. On the first day of
each subsequent month, s1 adds b members while s2 multiplies its current numbers by a constant
factor r. Both the societies have the same number of members on July 2, 2004. If b = 10.5n, what is
the value of r?
a. 2.0 b. 1.9 c. 1.8 d. 1.7

67. Suppose n is an integer such that the sum of digits on n is 2, and 1010 < n 10n. The number of
different values of n is
a. 11 b. 10 c. 9 d. 8

1
68. Let y =
1
2+
1
3+
1
2+
3 + ...
What is the value of y?
11 + 3 11 − 3 15 + 3 15 − 3
a. b. c. d.
2 2 2 2

69. The reminder, when (1523 + 2323) is divided by 19, is


a. 4 b. 15 c. 0 d. 18

CAT 2005

3065 – 2965
70. If R = , then
3064 + 2964
a. 0 < R ≤ 0.1 b. 0 < R ≤ 0.5 c. 0.5 < R ≤ 0.1 d.R > 1.0

71. If x = (163 + 173 + 183 + 193), then x divided by 70 leaves a remainder of


a. 0 b. 1 c. 69 d. 35

72. let n! = 1 × 2 × 3 × … × n for integer n ≥ 1 . If p = 1! + (2 × 2!) + (3 × 3!) + … + (10 × 10!), then p +


2 when divided by 11! Leaves a remainder of
a.10 b. 0 c. 7 d. 1

Page 8 Number System


73. The digits of a three-digit number A are written in the reverse order to form another three-digit
number B. If B > A and B-A is perfectly divisible by 7, then which of the following is necessarily
true?
a. 100 < A < 299 b. 106 < A < 305 c. 112 < A < 311 d. 118< A < 317

74. The rightmost non-zero digits of the number 302720 is


a. 1 b. 3 c. 7 d. 9

75. For a positive integer n, let pn denote the product of the digits of n and sn denote the sum of the
digits of n. The number of integers between 10 and 1000 for which pn + sn = n is
a.81 b. 16 c. 18 d. 9
76. Let S be a set of positive integers such that every element n of S satisfies the conditions
(a) 1000 ≤ n ≤ 1200
(b) every digit in n is odd
Then how many elements of S are divisible by 3?
a. 9 b. 10 c. 11 d. 12

CAT 2006

77. If x = – 0.5, then which of the following has the smallest value?
1 1 1 1
1. 2. 3. 4. 2X 5.
2x x x 2 −x

1 1 1 1 1
78. Which among 2 , 3 , , 6 and is the largest?
2 3 44 6 1212

1. 21/2 2. 31/3 3. 41/4 4. 61/6 5. 12/12

79. A group of 630 children is arranged in rows for a group photograph session. Each row contains three
fewer children than the row in front of it. What number of rows is not possible?
1. 3 2. 4 3. 5 4. 6 5. 7

80. The sum of four consecutive two-digit odd numbers, when divided by 10, becomes a perfect square.
Which of the following can possibly be one of these four numbers?
1. 21 2. 25 3. 41 4. 67 5. 73

81. When you reverse the digits of the number 13, the number increases by 18. How many other two-
digit numbers increase by 18 when their digits are reversed?
1. 5 2. 6 3. 7 4. 8 5. 10

Number System Page 9


Answers and Explanations
CAT 1998 6. c Let us find some of the smaller multiples of 125. They
are 125, 250, 375, 500, 625, 750, 875, 1000 ...
1. c If n3 is odd, then n should also be odd. Hence, n2 A five-digit number is divisible by 125, if the last three
should also be odd. And n2 will again be odd and not digits are divisible by 125. So the possibilities are 375
even. So only I and II are true. and 875, 5 should come in unit’s place, and 7 should
come in ten’s place. Thousand’s place should contain
2. b Since MPB is a three-digit number, and also the square 3 or 8. We can do it in 2! ways. Remaining first two
of a two-digit number, it can have a maximum value of digits, we can arrange in 2! ways. So we can have 2!
961 viz. 312. This means that the number BE should be × 2! = 4 such numbers.
less than or equal to 31. So B can only take the values There are: 23875, 32875, 28375, 82375.
0, 1, 2 and 3. Since the last digit of MPB is also B, it can
only be 0 or 1 (as none of the squares end in 2 or 3). 7. b Since 2 has a cyclicity of 4,
The only squares that end in 0 are 100, 400 and 900. i.e. 21 = 2, 22 = 4, 23 = 8, 24 = 16, 25 = 32, 26 = 64 ..., the
But for this to occur the last digit of BE also has to be last digits (2, 4, 8, 6) are in four cycles.
0. Since E and B are distinct integers, both of them 51
cannot be 0. Hence, B has to be 1. BE can be a number ∴ On dividing 4 , we get the remainder as 3.
between 11 and 19 (as we have also ruled out 10),
with its square also ending in 1. Hence, the number BE ∴ The last digit has to be 23 = 8
can only be 11 or 19. 112 = 121. This is not possible as
this will mean that M is also equal to 1. Hence, our Shortcut:
actual numbers are 192 = 361. Hence, M = 3. Since cyclicity of the power of 2 is 4, so 251 can be
written in 24(12) + 3 or unit digit will be 23 = 8.
3. c The maximum and the minimum five-digit numbers that
can be formed using only 0, 1, 2, 3, 4 exactly once are 8. a The number formed by the last 3 digits of the main
43210 and 10234 respectively. The difference number is 354. The remainder is 2 if we divide 354 by
between them is 43210 – 10234 = 32976. 8. So the remainder of the main number is also 2 if we
divide it by 8.
4. a The best way to solve this question is the method of
simulation, i.e. take a number which when divided by CAT 1999
899 gives a remainder of 63. The smallest such number
is (899 + 63) = 972. 972, when divided by 29 gives a 9. a (ab)2 = ccb, the greatest possible value of ‘ab’ to be
remainder of 5. Hence, the answer is 5. 31. Since 312 = 961 and since ccb > 300, 300 < ccb <
Students, please note that 899 itself is divisible by 29. 961, so 18 < ab < 31. So the possible value of ab
Hence, the required remainder is the same as obtained which statisfies (ab)2 = ccb is 21. So 212 = 441,
by dividing 63 by 29, i.e. 5. ∴ a = 2, b = 1, c = 4.

Shortcut: 10. b Note: 342 = 73 – 1. On further simplification we get,


Since 899 is divisible by 29, so you can directly divide
(73 )28 34328 (342 + 1)
28
63 = = =
the remainder of 63 by 29, so will give 5 as a 342 342 342
29
remainder, option (a). 342 N + 1 1
= =
342 342
5. b Note that the difference between the divisors and the Hence, remainder = 1, optiion (b).
remainders is constant.
2–1=3–2=4–3=5–4=6–5=1 11. a Use the method of simulation, viz. take any sample
In such a case, the required number will always be values of x and verify that n is both odd as well as a
[a multiple of LCM of (2, 3, 4, 5, 6) – (The constant perfect square.
difference)].
LCM of (2, 3, 4, 5, 6) = 60 12. b If there are n numbers, the function h has to be
Hence, the required number will be 60n – 1. performed one time less.
Thus, we can see that the smallest such number is
(60 × 1) – 1 = 59 13. c The square root is 11111111.
The second smallest is (60 × 2) – 1 = 119
So between 1 and 100, there is only one such number, 14. d None of the statements are true.
viz. 59.

Number System Page 1


15. a S1 remains the same, but S2 changes. 26. c There are two possible cases. The number 9 comes
at the end, or it comes at position 4, 5, or 6.
16. d x must be equal to the greatest difference in the value For the first case, the number would look like:
of numbers of S1 and S2.
9
635 ... ... 9.
674
CAT 2000
In both these cases, the blanks can be occupied by
a1 a2 any of the available 9 digits (0, 1, 2, ..., 8).
17. b 99 × D = a1a2. Hence, D= . So D must be multiplied
99 Thus, total possible numbers would be
by 198 as 198 is a multiple of 99. 2 × (9 × 9 × 9) = 1458. For the second case, the
number 9 can occupy any of the given position 4, 5, or
18. b Use any 7 consecutive numbers to check the 6, and there shall be an odd number at position 7.
answers. Thus, the total number of ways shall be
2[3(9 × 9 × 4)] = 1944. Hence, answer is 3402.
n=
(1 + 2 + 3 + 4 + 5 ) = 3 , average of 7 integers is
5 27. d N can be written either (54 + 1)3 + (18 – 1)3 – 723 or
(51+ 4 )3 + 173 – (68 + 4)3 .
k=
(1 + 2 + 3 + 4 + 5 + 6 + 7 ) = 4 . The first form is divisible by 3, and the second by 17.
7
So k = n + 1. 28. c
Alternately, the average of the first 5 terms is the
middle term which is third term, and the average of the 1 2 19 82
first 7 terms is the middle term which is the fourth 1 2 16 5 – 2
term. Hence, it is one more than the previous average. 1 2 13 – 9
1 – 1
19. d Numbers which are divisible by 3 (between 100 and
200) are 33. Numbers which are divisible by 21, i.e. The answer is 1192.
LCM of 7 and 3 (between 100 and 200) are 5. Out of
the 33 numbers divisible by 3, 17 are even and 16 are
odd. Out of the 5 numbers divisible by 7, three are
CAT 2001
odd. Hence, the number of odd numbers divisible by 3
but not by 7 is (16 – 3) = 13. 29. a Use the answer choices and the fact that:
Odd × Odd = Odd
20. d Take any three odd and positive numbers and check Odd × Even = Even
this out. Even × Even = Even

21. a There is only one 5 and one 2 in the set of prime 30. a Let the four-digit number be abcd.
numbers. Hence, there would be only one zero at the a+b=c+d ... (i)
end of the resultant product. b + d = 2(a + c) ... (ii)
a+d=c ... (iii)
22. c If the sides of the triangle are a, b and c, then From (i) and (iii), b = 2d
a + b > c. Given a + b + c = 14. From (i) and (ii), 3b = 4c + d
Then the sides can be (4, 4, 6), (5, 5, 4), (6, 5, 3) ⇒ 3(2d) = 4c + d
and (6, 6, 2). Hence, four triangles are possible. ⇒ 5d = 4c
5
23. c N = 1421 × 1423 × 1425. When divided by 12, it shall ⇒ c= d
4
(1416 + 5 ) × (1416 + 7 ) × (1416 + 9 )
look like  . Now d can be 4 or 8.
12 But if d = 8, then c = 10 not possible.
Now the remainder will be governed by the term So d = 4 which gives c = 5.
5 × 7 × 9, which when divided by 12 leaves the
remainder 3. 31. d Let the number be x.
Increase in product = 53x – 35x = 18x
24. d Let r be the remainder. Then 34041 – r and 32506 – r ⇒ 18x = 540 ⇒ x = 30
are perfectly divisible by n. Hence, their difference Hence new product = 53 × 30 = 1590.
should also be divisible by the same.
(34041 – r) – (32506 – r) = 1535, which is divisible by 32. c The value of y would be negative and the value of x
only 307. would be positive from the inequalities given in the
question.
25. a Each term has to be either 1 or –1. Therefore, from (a), y becomes positive. The value of
Hence, if the sum of n such terms is 0, then n is even. xy2 would be positive and will not be the minimum.
From (b) and (c), x2y and 5xy would give negative
values but we do not know which would be the
minimum.

Page 2 Number System


On comparing (a) and (c), we find that c = 6, c = 8 (Not possible)
x2 < 5x in 2 < x < 3. c = 10, e = 10 (Not possible since both c and e cannot
be 10)
∴ x2 y > 5xy [Since y is negative.] From (ii), we have c = 2, 4, 6, 10.
∴ 5xy would give the minimum value. For c = 2, e = 3 (Not possible)
c = 4, e = 4 (Not possible)
33. a The product of 44 and 11 is 484. c = 6, e = 5 (Possible)
If base is x, then 3411 c = 10, e = 7 (Not possible)
Considering the possibility from B that c = 6 and
= 3x3 + 4 x2 + 1x1 + 4 × x0 = 484 e = 5 means e + a = 4
⇒ 3 x3 + 4 x2 + x = 480 ⇒ a = –1 (Not possible)
This equation is satisfied only when x = 5. Hence, only possibility is b = 10, d = 5, c = 2, e = 6.
So base is 5. e + a = 10 ⇒ a = 4
In decimal system, the number 3111 can be written
39. c The last two digits can be 12, 16, 24, 32, 36, 52, 56,
3 × 53 + 1 × 5 2 + 1 × 51 + 1 × 50 = 406 and 64, i.e. 8 possibilites
Remaining digits can be chosen in 4
P3 = 24 ways.
34. c Let the total number of pages in the book be n.
Hence, total number of such five-digit numbers
n

∑i + x = 1000
= 24 × 8 = 192.
Let page number x be repeated. Then
i =1
CAT 2002
n(n + 1)
+ x = 1000 40. c Total possible arrangements = 10 × 9 × 8
2 Now 3 numbers can be arranged among themselves
n(n + 1) in 3! ways = 6 ways
Thus, ≤ 1000 gives n = 44 Given condition is satisfied by only 1 out of 6 ways.
2 Hence, the required number of arrangements
n(n + 1) 10 × 9 × 8
Since = 990 (for n = 44). = = 120
2 6
Hence x = 10.
41. b Check choices
35. c Take a = b = c = d = 1. Choice (b) 54 ⇒ S = (5 + 4)2 = 81
⇒ D – S = 81 – 54 = 27. Hence, the number = 54
36. a Let the highest number be n and x be the number
erased. 42. d Let the number of gold coins = x + y
n(n + 1) 48(x – y) = X2 – Y2
–x 48(x – y) = (x – y)(x + y) ⇒ x + y = 48
2 7 602 .
Then = 35 = Hence the correct choice would be none of these.
(n – 1) 17 17
Hence, n = 69 and x = 7 satisfy the above conditions.
n2 + n
43. d 575 = –x
37. d a = b2 – b, b ≥ 4 2
a2 – 2a = (b2 – b)2 – 2(b2 – b)
= (b2 – b)(b2 – b – 2) 1150 = n2 + n – 2x
Using different values to b ≥ 4 and we find that it is n(n + 1) ≥ 1150
divisible by 15, 20, 24.
Hence all of these is the right answer. n2 + n ≥ 1150
The smallest value for it is n = 34.
38. b From II, b = 2d For n = 34
Hence, b = 10, d = 5 or b = 4, d = 2 40 = 2x ⇒ x = 20
From III, e + a = 10 or e + a = 4
From I, a + c = e or e – a = c
(2 )
64
From III and I, we get 2e = 10 + c or 2e = 4 + c 44. a 4
= (17 – 1)64 = 17n + (−1)64 = 17n + 1
c
⇒ e=5+ ... (i) Hence, remainder = 1
2
45. b Because each word is lit for a second,
c
or e = 2 + ... (ii)
2 5 17 41   7 21 49 
LCM  + 1, + 1, + 1 = LCM  , , 
From (i), we can take c = 2, 4, 6, 10. 2 4 8  2 4 8 
For c = 2, e = 6
LCM(7, 21, 49) 49 × 3
c = 4, e = 7 (Not possible) = = 73.5 s
HCF ( 2, 4, 8) 2

Number System Page 3


54. c If y = 2 (it cannot be 0 or 1), then x can take 1 value
 9 27 36  HCF(9, 27, 36) = 9
 = LCM (2, 4, 5)
46. d HCF  , , lb and z can take 2 values.
2 4 5  20 Thus with y = 2, a total of 1 × 2 = 2 numbers can be
= Weight of each piece formed. With y = 3, 2 × 3 = 6 numbers can be formed.
Total weight = 18.45 lb Similarly checking for all values of y from 2 to 9 and
adding up we get the answer as 240.
18.45 ×20
Maximum number of guests = = 41
9 55. c The best way to do this is to take some value and
verify.
47. d 3(4(7x + 4) + 1) + 2 = 84x + 53
1
Therefore, remainder is 53. E.g. 2, and 1. Thus, n = 3 and the sum of the three
2
numbers = 3.5.
48. d um + vm = w m Thus options 1, 2 and 4 get eliminated.
u2 + v 2 = w 2 Alternative method:
Taking Pythagorean triplet 3, 4 and 5, we see Let the n positive numbers be a1, a2, a3 … an
m < min (u, v, w) a1, a2, a3 … an = 1
Also 1' + 2' = 3' and hence m ≤ min (u, v, w) We know that AM ≥ GM
1
49. d 76n – 66n Hence (a1 + a2 + a3 + … + an ) ≥ (a1a2 …an )1/ n
n
Put n = 1
or (a1 + a2 + a3 + … an) ≥ n
76 – 66 = (73 – 63 )(73 + 63 )
56. b From 12 to 40, there are 7 prime number, i.e. 13, 17,
This is a multiple of 73 – 63 = 127 and 73 + 63 = 559 19, 23, 29, 31, 37, which is not divisible by (n–1)!
and 7 + 6 = 13
CAT 2003 (Retest)
50. c Number of ways for single digit = 2
2 digits = 2 × 3 = 6 57. b MDCCLXXXVII = 1000 + 500 + 100 + 100 + 50 + 10 + 10
3 digits = 2 × 3 × 3 = 18 + 10 + 5 + 1 + 1 = 1787
4 digits = 2 × 3 × 3 × 3 = 54
5 digits = 2 × 3 × 3 × 3 × 3 = 162 58. a MCMXCIX = 1000 + (1000 – 100) + (100 – 10) + (10 –
6 digits = 2 × 3 × 3 × 3 × 3 × 3 = 486 1) = 1000 + 900 + 90 + 9 = 1999
Total = 728
59. c (I) MCMLXXV = 1000 + (1000 – 100) + 50 + 10 + 10
CAT 2003 (leaked) + 5 = 1975
(II) MCMXCV = 1000 + (1000 – 100) + (100 – 10) + 5
51. c There are 101 integers in all, of which 51 are even. = 1995
From 100 to 200, there are 14 multiples of 7, of which (III) MVD = 1000 + (500 – 5) = 1495
7 are even. (IV) MVM = 1000 + (1000 – 5) = 1995
There are 11 multiples of 9, of which 6 are even. Therefore, the answer is (II) and (IV), i.e. option (c).
But there is one integer (i.e. 126) that is a multiple of
both 7 and 9 and also even. 60. b Such numbers are 10, 17, …, 94.
Hence the answer is (51 – 7 – 6 + 1) = 39 These numbers are in AP. There are 13 numbers.
10 + 94
52. d Since the last digit in base 2, 3 and 5 is 1, the number ∴ Sum = × 13
2
should be such that on dividing by either 2, 3 or 5 we
should get a remainder 1. The smallest such number is = 52 × 13 = 676
31. The next set of numbers are 61, 91.
Among these only 31 and 91 are a part of the answer 61. c Total codes which can be formed = 9 × 9 = 81.
choices. (Distinct digit codes)
The digits which can confuse are 1, 6, 8, 9, from
Among these, (31)10 = (11111)2 = (1011)3 = (111)5 these digit we can form the codes = 4 × 3 = 12
Thus, all three forms have leading digit 1. Out of these 12 codes two numbers 69 and 96 will not
Hence the answer is 91. create confusion.
Therefore, (12 – 2) = 10 codes will create a confusion.
53. a Solution cannot be found by using only Statement A Therefore, total codes without confusion
since b can take any even number 2, 4, 6…. = 81 – 10 = 71.
But we can arrive at solution by using statement B
alone.
If b > 16, say b = 17
Hence 244 < (16 + 1)11
244 < (24 + 1)11

Page 4 Number System


67. a We have
496
62. d Remainder when (1) 1010 < n < 1011
6 (2) Sum of the digits for 'n' = 2
Let’s come down to basic property of dividing the Clearly-
power of 4 by 6, i.e. (n)min = 10000000001 (1 followed by 9 zeros and
41 finally 1)
=4 Obviously, we can form 10 such numbers by shifting
6
'1' by one place from right to left again and again.
42
=4 Again, there is another possibility for 'n'
6 n = 20000000000
So finally : No. of different values for n = 10 + 1 = 11
43
=4 ans.
6

44 68. d y=
1
=4 1
6 2+
3+ y
Hence, any power of 4 when divided by 6 leaves a
remainder of 4. 3+y
⇒y=
7 + 2y
63. d Let n = 6
Therefore, n = 6 ≈ 2.4 ⇒ 2y 2 + 6y – 3 = 0
Therefore, divisors of 6 are 1, 2, 3. –6 ± 36 + 24
⇒y=
If we take 2 as divisor, then n > 2 >1. 4
Statement A is true.
–6 ± 60 –3 ± 15
If we take 3 as divisor, then 6 > 3 > 2.4, i.e. n > 3 > n. = =
4 2
Therefore, statement B is true.
Since 'y' is a +ve number, therefore:
64. a As any prime number greater than 3 can be expressed
in the form 6n ± 1 , minimum difference between three 15 –3
y= ans.
consecutive prime numbers will be 2 and 4. The values 2
that satisfy the given conditions are only 3, 5 and 7,
i.e. only one set is possible.
69. c 1523 = (19 – 4)23 = 19x + (–4)23 where x is a natural
65. d a = 6b = 12c and 2b = 9d = 12e. number.
Dividing the first equations by 12 and second by 36,
2323 = (19 + 4)23 = 19y + (4)23 where y is a natural
a b c b d e
we get = = and = = number.
12 2 1 18 4 3
1523 + 2323 = 19 ( x + y ) + 423 + (–4)23
a b c b d e
i.e. = = and = = = 19 ( x +y)
108 18 9 18 4 3
a b c d e
∴ = = = =
108 18 9 4 3 CAT 2005
∴ a : b : c : d : e = 108 : 18 : 9 : 4 : 3.
c 9 3065 – 29 65
∴ = is not an integer. 70. d >1
d 4 3064 + 2964
as 3065 – 2965 > 3064 + 2964
CAT 2004 3064 (30 – 1) > 2964 (29 + 1)
3064 × 29 > 2964 × 30
66. a There will be an increase of 6 times. 3063 > 2963
No. of members s1 will be in A.P. Hence option d.
On July 2nd , 2004, s1 will have n + 6 b members
= n + 6 × 10.5 n 71. a x = 163 + 173 + 183 + 193 is even number
= 64n Therefore 2 divides x.
No. of members in s2 will be in G.P a3 + b3 = (a + b) (a2 – ab – b2)
On July 2nd, 2004 Number of members in s2 ⇒ a + b always divides
= nr6 Therefore 163 + 193 is divisible by 35
They are equal, Hence 64 n = nr6 183 + 173 is divisible by 35
Hence x is divisible by 70.
⇒ 64 = r 6 ⇒ r = 2
Hence option a.

Number System Page 5


72. d If p = 1! = 1
−1/ 2 1
Then p + 2 = 3 when divided by 2! remainder will be (4) 2 =
1. 2
If p = 1! + 2 × 2! = 5 78. 2 LCM of 2, 3, 4, 6, 12 = 12
Then p + 2 = 7 when divided by 3! remainder is still
12 6 12 4 12 3 12 2 12
1. 2 3 4 6 121
Hence p = 1! + (2 × 2!) + (3 × 3!) + … + (10 × 10!) ∴ 34 is greatest
when divided by 11! leaves remainder 1 Note: n1/n is maximum when n = e (2.718). Among the
options n = 3 is closest to the value of e.
Alternative method:
P = 1 + 2.2! + 3.3!+ ….10.10! 79. 4 Let the no. of students in front row be x.
= (2 –1)1! + ( 3 – 1)2! + (4 – 1)3! + ….(11 – 1)10! So, the no. of students in next rows be x – 3,
=2! – 1! + 3! – 2! + ….. 11! –10! x - 6, x – 9…. so on
= 1 + 11! If n i.e. no. of rows be 3 then no. of students
Hence the remainder is 1. x + (x – 3) + (x – 6) = 630
3x = 639
73. b Let A = 100 x + 10y + z x = 213
⇒ B = 100z + 10y + x So possible similarly n = 4
B - A = 99(z - x) x + (x – 3) + (x – 6) + (x - 9) = 630
For B - A to be divided by 7, z - x has to be divisible 4x – 18 = 630
by 7. Only possibility is z = 9, x = 2.
Biggest number A can be 299 648
x= = 162
Option b. 4
If n = 5
74. a ((30) )
4 680 (4x – 18) + (x - 12) = 630
5x – 30 = 630
Hence the right most non-zero digit is 1. x = 120
Again possible.
75. d 10 < n < 1000 If n = 6
Let n is two digit number. (5x - 30) + (x - 15) = 630
n = 10a + b ⇒ pn = ab, sn = a + b 6x - 45 = 630
Then ab + a + b = 10a + b 6x = 675
⇒ ab = 9a ⇒ b = 9 x ≠ Integer
There are 9 such numbers 19, 29, 33, … 99 Hence n ≠ 6
Then Let n is three digit number
⇒ n = 100a + 10b + c ⇒ pn = abc, sn= a + b + c 80. 3 By options checking option (3), four consecutive odd
then abc + a + b + c = 100a + 10b + c numbers are 37, 39, 41 and 43. The sum of these 4
⇒ abc = 99a + 9b numbers is 160.
When divided by 10, we get 16 which is a perfect
b
⇒ bc = 99 + 9 square.
a ∴ 41 is one of the odd numbers.
But the maximum value for bc = 81
And RHS is more than 99. Hence no such number is 81. 2 Let the number be 10x + y so when number is reversed
possible. the number because 10y + x. So, the number increases
Hence option d. by 18
Hence (10y + x) - (10x + y) = 9 (y - x) = 18
76. a The 100th and 1000th position value will be only 1 .
y-x=2
Now the possibility of unit and tens digits are
So, the possible pairs of (x, y) is (3, 1) (4, 2) (5, 3) (6,
(1, 3), (1, 9), (3, 1), (3, 7), (5, 5), (7, 3), (7, 9), (9, 1),
4), (7, 5) (8, 6) (9, 7)
(9, 7). But we want the number other than 13 so, there are 6
possible numbers are there i.e. 24, 35, 46, 57, 68, 79.
CAT 2006 So total possible numbers are 6.

−1
77. 2 Go by option, put x =
2

−2 1
(1) 2 =
4

1 1
(2) ⇒ = −2
x −1/ 2

1 1
⇒ =4
(3) x 2
( −1/ 2 )2

Page 6 Number System

You might also like